Last visit was: 26 Apr 2024, 19:51 It is currently 26 Apr 2024, 19:51

Close
GMAT Club Daily Prep
Thank you for using the timer - this advanced tool can estimate your performance and suggest more practice questions. We have subscribed you to Daily Prep Questions via email.

Customized
for You

we will pick new questions that match your level based on your Timer History

Track
Your Progress

every week, we’ll send you an estimated GMAT score based on your performance

Practice
Pays

we will pick new questions that match your level based on your Timer History
Not interested in getting valuable practice questions and articles delivered to your email? No problem, unsubscribe here.
Close
Request Expert Reply
Confirm Cancel
SORT BY:
Date
User avatar
Intern
Intern
Joined: 22 Jun 2010
Posts: 18
Own Kudos [?]: 811 [50]
Given Kudos: 1
Send PM
Most Helpful Reply
Math Expert
Joined: 02 Sep 2009
Posts: 92948
Own Kudos [?]: 619253 [12]
Given Kudos: 81609
Send PM
General Discussion
User avatar
Intern
Intern
Joined: 15 Aug 2010
Posts: 11
Own Kudos [?]: 42 [0]
Given Kudos: 0
Location: Mumbai
Schools:Class of 2008, IIM Ahmedabad
Send PM
User avatar
Manager
Manager
Joined: 22 Feb 2009
Posts: 109
Own Kudos [?]: 526 [0]
Given Kudos: 148
Send PM
Re: 2001, the closing price of stock A and stock B are the same. [#permalink]
4gmatmumbai wrote:
Hi,

Stmt 1:

Let the closing stock price of both A and B (in 2001) be Rs 100.

Case 1:

Let the closing stock price of B at the end of 2002 be Rs 200. Then, A is Rs 216.

Price increase of stock A = Rs 116
Price increase of stock B = Rs 100

% by which price increase in stock A is greater than that in stock B =
((116 - 100) / 100) * 100 = 16%

Case 2:

Let the closing stock price of B at the end of 2002 be Rs 300. Then, A is Rs 324.

Price increase of stock A = Rs 224
Price increase of stock B = Rs 200

% by which price increase in stock A is greater than that in stock B =
((224 - 200) / 200) * 100 = 12%

Hence, statement 1 is not sufficient. Hope this clarifies.

I guess the reason why stmt 2 is not sufficient is fairly straightforward.

Thanks.


I think the try and error will consume a lot of time. I prefer the solution of Brunel :)
Tutor
Joined: 17 Jul 2019
Posts: 1304
Own Kudos [?]: 2287 [3]
Given Kudos: 66
Location: Canada
GMAT 1: 780 Q51 V45
GMAT 2: 780 Q50 V47
GMAT 3: 770 Q50 V45
Send PM
Re: 2001, the closing price of stock A and stock B are the same. [#permalink]
1
Kudos
2
Bookmarks
Expert Reply
Video solution from Quant Reasoning starts at 15:40 here:
Intern
Intern
Joined: 12 Jan 2022
Posts: 2
Own Kudos [?]: 1 [1]
Given Kudos: 16
Send PM
Re: 2001, the closing price of stock A and stock B are the same. [#permalink]
1
Kudos
A simply way to solve this question is by assuming values

Statement 1 is not sufficient because we have no information about B and we can get more than 1 answer

Statement 2 is not sufficient because again, we we have no information about B.

When we combine both the statements we get

Assume the starting value of both the stocks is 100.
Let the increase in stock B be x

Now,
1.15(100)=1.08(100+x)
115=108+1.08x
7=1.08x
x=7/1.08
x=6.48

Hope this was helpful

You can follow similar approach in such questions

Aryan Chandel
+91 8169397962
Intern
Intern
Joined: 24 Jul 2013
Posts: 20
Own Kudos [?]: 3 [0]
Given Kudos: 111
Send PM
Re: 2001, the closing price of stock A and stock B are the same. [#permalink]

    stmt 1
    A B
    2001 X X
    2002 108 100

    Price increase for A = (108-X)/X
    Price increase for B = (100-X)/X

    Question asks : By what percent was the price increase of A greater than the price increase of B
    (Price increase for A - Price increase for B)/Price increase for B

    Not Sufficient (X is not known)

    stmt 2:
    Price increase for A = 15%, We dont know anything abt B --> Not Sufficient

    Combining 1 and 2

    15=(108-X)/X
    108=16X

    Stmt 1 + stmt 2 together sufficient








User avatar
Non-Human User
Joined: 09 Sep 2013
Posts: 32691
Own Kudos [?]: 822 [0]
Given Kudos: 0
Send PM
Re: 2001, the closing price of stock A and stock B are the same. [#permalink]
Hello from the GMAT Club BumpBot!

Thanks to another GMAT Club member, I have just discovered this valuable topic, yet it had no discussion for over a year. I am now bumping it up - doing my job. I think you may find it valuable (esp those replies with Kudos).

Want to see all other topics I dig out? Follow me (click follow button on profile). You will receive a summary of all topics I bump in your profile area as well as via email.
GMAT Club Bot
Re: 2001, the closing price of stock A and stock B are the same. [#permalink]
Moderator:
Math Expert
92948 posts

Powered by phpBB © phpBB Group | Emoji artwork provided by EmojiOne